6
$\begingroup$

In a quadratic extension $\mathbb{Q}(\sqrt{d})$of $\mathbb{Q}$ it is clear that 2 ramifies if and only if $d\equiv 2,3\mod 4$ (easy to see if you compute the discriminant). But if I take a relative quadratic extension, to make it simple let's say $L=K(\sqrt{\pi})$ where $K$ is an arbitrary number field and $\pi$ is an irreducible element of the ring of integers of $K$, when does a dyadic prime ramify from $K$ to $L$?

Certainly $\pi$ itself ramifies in this case, that's easy to see without knowing the discriminant or even ring of integers. But how do I know when primes above $2$ are ramifying, i.e. being contributed to the discriminant?

In general, I know that computing the discriminant (or ring of integers) of such a field, is computationally complex. But is there a known congruence condition on $\pi$ or some algebraic way to decide when primes above 2 ramify?

$\endgroup$
4
  • 3
    $\begingroup$ $L=K(\sqrt{\mathfrak{p}})$ makes no sense. There is no such thing as extending by the "square-root of an ideal" (unless you define it here). You can extend by the square-root of an element or, more generally, by the root of a polynomial with coefficients lying in the base field. So please clarify. $\endgroup$
    – GH from MO
    Mar 16, 2014 at 11:26
  • 1
    $\begingroup$ Thanks, right, I just want $\mathfrak{p}$ to be a prime element in the base field $K$. $\endgroup$
    – user48331
    Mar 16, 2014 at 11:41
  • 3
    $\begingroup$ @user48331 Probably you mean that $\pi$ is an irreducible element of the ring of integers of $K$, and you're looking at the ring of integers of $K(\sqrt{\pi})$. But the set of irreducible elements is not so nice in general. So I think that your attempted simplification is actually confusing the issue. Why not just ask: Is there a congruence condition on $\delta\in O_K$ that determines whether primes above 2 are ramified in $O_{K(\sqrt{\delta})}$? $\endgroup$ Mar 16, 2014 at 13:09
  • 1
    $\begingroup$ This is a purely local question, easily answered, but someone else will do it before I get back — gotta run. $\endgroup$
    – Lubin
    Mar 16, 2014 at 13:11

2 Answers 2

8
$\begingroup$

There's an appendix on Kummer theory to Milne's class field theory notes (available here) that gives an answer to this question (EDIT: in some cases). Assuming that $K$ contains a primitive $p$th root of unity $\zeta$, $\pi = (\zeta - 1)$ and $a \in K^{\times}$ is such that

$\bullet$ $a$ is not a $p$th power in $K$,

$\bullet$ $a$ is relatively prime to $p$, and

$\bullet$ $X^{p} \equiv a \pmod{p \pi}$ has a nonzero solution $X \in \mathcal{O}_{K}$,

then none of the primes $\mathfrak{p}$ above $p$ ramify in $K[a^{1/p}]/K$.(This is Remark A.6 on page 223.)

Milne doesn't give a proof, instead referring to exercises in Washington's Cyclotomic Fields text (pages 182 and 183 there), and Cassels and Frohlich.

$\endgroup$
7
  • $\begingroup$ Many thanks, that is precisely what I was looking for. $\endgroup$
    – user48331
    Mar 17, 2014 at 9:22
  • $\begingroup$ No hypothesis on $a$ being a unit? After all, with $a=8$, $\mathbb Q(\sqrt8)$ is ramified over $\mathbb Q$ but $x^2\equiv8\pmod4$ has a perfectly good solution in $\mathbb Q$, namely zero. $\endgroup$
    – Lubin
    Mar 17, 2014 at 12:56
  • $\begingroup$ The Milne notes do specify that the solution $x$ must be nonzero. But in general this is another good question. What happens when $a$ is even? $\endgroup$
    – user48331
    Mar 17, 2014 at 14:17
  • $\begingroup$ There were more hypotheses from Milne that I missed - my answer has been edited to be correct. (The version is somewhat less helpful, however.) $\endgroup$ Mar 17, 2014 at 17:36
  • 1
    $\begingroup$ Some care is needed: $\mathbb{Q}(\sqrt{20})/\mathbb{Q}$ is unramified at $2$. Given $a \in K_{\mathfrak{p}}$, one can write $a = \pi^{k} a'$, where $\pi$ is a generator of the unique maximal ideal of $\mathcal{O}_{K_{\mathfrak{p}}}$ and $a' \in \mathcal{O}_{K_{\mathfrak{p}}}^{\times}$. If $k$ is odd, then $K_{\mathfrak{p}}(\sqrt{a})$ is ramified. If $k$ is even, then $K_{\mathfrak{p}}(\sqrt{a}) = K_{\mathfrak{p}}(\sqrt{a'})$ (reducing to an "odd" value of $a$). $\endgroup$ Mar 18, 2014 at 12:09
6
$\begingroup$

While I was away I realized that I didn’t have a complete answer to this question. But the key to finding whether $K(\sqrt\delta)\supset K$ is ramified over $2$ is to take a prime $\mathfrak q$ of $K$ above $2$, and consider $V=v_{\mathfrak q}$, the (additive) valuation at $\mathfrak q$ and $V(\delta-1)$. I’ll normalize $V$ so that $V(2)=1$.

Now, in view of the fact that the binomial expansion $(1+4t)^{1/2}$ has its coefficients in $\mathbb Z$, you see that if $V(\delta-1)>2$, the prime $\mathfrak q$ splits in the extension. Similarly, if $\delta=1+4u$ for a $\mathfrak q$-unit $u$, you see directly that the extension is unramified: may or may not split.

The same kind of computation shows that if $V(\delta-1)<2$, $V(\sqrt\delta-1)=\frac12V(\delta-1)$. In case $V(\delta)=m/e$ with $m$ odd, and where the local ramification is $e$, you’re home free: the extension is definitely ramified. But I don’t see offhand how you can exclude the possibility that $m$ is even, and there I’m not so sure.

$\endgroup$

Your Answer

By clicking “Post Your Answer”, you agree to our terms of service and acknowledge you have read our privacy policy.

Not the answer you're looking for? Browse other questions tagged or ask your own question.